Steady State output for Wave Input

Click For Summary
The discussion focuses on finding the steady state output yss(t) for the input u(t)=t-π using an infinite sum of sinusoids. The transfer function and relevant equations are provided, with G(i) determined as 1. A Fourier Series expansion yielding a sum of -2sin(kt)/k has been previously established. The main challenge is incorporating these values into the steady state formula, with a request for tips on inputting mathematical expressions. The conversation emphasizes the need for clarity in applying the Fourier Series to the steady state output calculation.
soccer4life
Messages
13
Reaction score
0
1. Problem Statement
Find the steady state output yss(t) for the input u(t)=t-π in terms of an infinite sum of sinusoids.
We are given the transfer function as:
help-png.png

2. Homework Equations

G(i) = ...
|G(ik)| = ...
Φ(ik) = ... (this is the angle)
yss(t) = βk||G(ik)|ei(kt+Φ(ik)) ***check that this is the correct formula please***

3. Attempt at Solution
I've found the following:
G(i)=1
|G(ik)| =
199549-3d8b3d65837a235ecb2c36e83fa44816.png
(Any tips/tricks on how to input fractions/square roots into PF would be greatly appreciated...)
Φ(ik) =
199551-ccce8e83197e676283b3561ae3b6c3be.png


Previously, the Fourier Series expansion was found, and is: the sum from 1 to infinity of Σ-2sin(kt)/k

I know that these values are right. However, I don't fully understand how to incorporate them into the steady state formula (assuming that my formula is correct)
 

Attachments

  • help-png.png
    help-png.png
    1.6 KB · Views: 770
  • 199549-3d8b3d65837a235ecb2c36e83fa44816.png
    199549-3d8b3d65837a235ecb2c36e83fa44816.png
    616 bytes · Views: 375
  • 199551-ccce8e83197e676283b3561ae3b6c3be.png
    199551-ccce8e83197e676283b3561ae3b6c3be.png
    677 bytes · Views: 349
Last edited by a moderator:
Physics news on Phys.org
Use the fact that ##\sin\theta = \frac{e^{i\theta}-e^{-i\theta}}{2i}##.
 
Question: A clock's minute hand has length 4 and its hour hand has length 3. What is the distance between the tips at the moment when it is increasing most rapidly?(Putnam Exam Question) Answer: Making assumption that both the hands moves at constant angular velocities, the answer is ## \sqrt{7} .## But don't you think this assumption is somewhat doubtful and wrong?

Similar threads

  • · Replies 1 ·
Replies
1
Views
2K
  • · Replies 1 ·
Replies
1
Views
2K
Replies
1
Views
2K
  • · Replies 10 ·
Replies
10
Views
3K
Replies
4
Views
4K
Replies
23
Views
6K
Replies
4
Views
6K
Replies
1
Views
15K
  • · Replies 3 ·
Replies
3
Views
4K
  • · Replies 9 ·
Replies
9
Views
5K